Can I get some help please? Time is limited!!!

Can I Get Some Help Please? Time Is Limited!!!

Answers

Answer 1
hi! i believe the third one cannot be solved by the square root method, because there’s an x without an exponent. hope this helped!

Related Questions

[tex]P=\frac{11}{40}+\frac{1}{4}-\frac{1}{20}[/tex]

Answers

Answer

p = 19/40

Please give brainliest

two rectangles have a scale factor of 4:1. If the area of the larger rectangle is 156cm^2, what is the area of the smaller rectangle?

A. 26.75 cm^2
B. 31 cm^2
C. 9.75 cm^2
D. 39 cm^2

Answers

Answer:

c=9.75cm^2

Step-by-step explanation:

156/16 because you are deivding each side by 4 and 4x4=16 and the area is 156

if you know the answer please help me ese ​

Answers

Answer:

Domain { 4,8,12,16,20}

Range { 1,2,3,4}

This is not a function because two of the inputs have more than one output

Step-by-step explanation:

The domain is the x values ( input)

Domain { 4,8,12,16,20}

The range is the y values (output)

Range { 1,2,3,4}

This is not a function because two of the inputs have more than one output

can some one pls help me with this i need help

Answers

Answer:

9/8 cup peanuts were used

The amounts are:

Correct

Correct

Incorrect

Question
2x + 3y = 11
x – 3y =1
What is the solution (x, y) to the given system of equations?
O (4,0)
O (4,1)
O (12,8)
O (12, 10)

Answers

Answer: (4, 1)

Step-by-step explanation:

Let's solve this by adding the equations

2x+3y=11

+x-3y=1

3x=12

Now we can solve

[tex]\frac{3x}{3} =\frac{12}{3} \\x=4[/tex]

Now that we know x=4 it can only be the first to options. Let's sub in the value of x to one of the original equations to find the value of y.

[tex]2(4)-3y=11\\8+3y=11\\8+3y-8=11-8\\\\3y=3\\\frac{3y}{3} =\frac{3}{3} \\y=1[/tex]

The answer is (4, 1)

Need help immediately
A sum of money is to be divided among A, B and C in the ratio 2:3:5. The smallest share amounts to $210.
Calculate:
i. The total sum of money to be shared
ii. C’s share
iii. B’s share​

Answers

Answer:

Step-by-step explanation:

If A, B, and C are in the ratio of 2:3:5 and they are each getting some amount of money in that ratio, then to find the total amount of money they split is found in Ax + Bx + Cx and we need to solve for x. What we are told is that the smallest share = 210 and obviously A is the smallest share. Therefore,

2x = 210 and x = 105.

The total sum = 210 + 3(105) + 5(105) so

Total sum = 210 + 315 + 525

Total sum = 1050 and

C's share is 525 and

B's share is 315

please help me immediately​

Answers

Answer:

zero of a polynomial can be defined as the points where the polynomial become 0 as a whole.

An equation formed with variables, exponents and coefficient together with operation and an equal sign is called polynomial equation.

I hope this will help you

A wire attached to the top of a pole reaches a stake in the ground 20 feet from the foot of the pole and makes an angle of 58° with the ground. Find the length of the wire.

Answers

it is a right triangle
cos 58 = 20/length of rope
length of rope = 20/cos 58 degrees
= 37.74 feet

2x-5y=-3
5x+2y=6
Is this a perpendicular lines ?

Answers

Perpendiculat Line is when both slopes of equation multiplying each others and equal to -1.

[tex] \large \boxed{m_1m_2 = - 1}[/tex]

For an easier way to understand, a perpendicular line has a negative reciprocal slope. For example if we are given the equation of y = 2x then the equation that is perpendicular to y = 2x would be y = (-1/2)x.

From both equations. We can either arrange in slope-intercepy form or use the slope formula which is m = -A/B when the equation is in Ax+By+C = 0 or Ax+By = C.

[tex] \large{ \begin{cases} 2x - 5y = - 3 \\ 5x + 2y = 6 \end{cases}}[/tex]

To find if these two lines are perpendicular to each others. Use the slope formula of -A/B.

[tex] \large{ \begin{cases} m_1 = - \frac{2}{ - 5} \\ m_2 = - \frac{5}{2} \end{cases}} \\ \large{ \begin{cases} m_1 = \frac{2}{ 5} \\ m_2 = - \frac{5}{2} \end{cases}}[/tex]

Now recall that it is perpendicular when one of them is negative reciprocal of one another (-5/2 is negative reciprocal of -(-2/5) = 2/5 and 2/5 is negative reciprocal of -5/2).

Or in definition of perpendicular lines, both slopes multiply and must equal to -1.

[tex] \large{( \frac{2}{5} )( - \frac{5}{2} ) = - 1} \\ \large{ - 1 = - 1}[/tex]

Thus the equation is true which makes both lines perpendicular to each others.

Answer

Both lines are perpendicular to each others.

Let me know if you have any doubts!

please help I will give brainlist​

Answers

Answer:

Your answer will be B

Step-by-step explanation:

Because you find the difference between 14 and 22 or the difference between 22 and 30 or the difference between 30 and 38. and so on then it will be 8

You then say ok 8 times x so for example 8×1 = 8 then you say ok 22 - 8= 22. So it will be 8x+14...

I hope this helped if not tell me please....❤

Answer: Option B (y=8x+14)

Step-by-step explanation:

To solve these kind of question we just simply keep on inserting the values of x and if we get the required value of y that is in the table that equation is our answer, For example lets take Option A,

y = 8x

If we insert x = 0 we get,

y = 8(0)

y=0

but in the table it says we get 14 so option A is incorrect

Like this we keep checking for more values and all the values of x satisfy our equation in Option B like this,

y=8x+14

y=8(0)+14

y=14

y=8x+14

y=8(1)+14

y=8+14

y=22

y=8x+14

y=8(2)+14

y=16+14

y=30

and so on. Now your answer would be Option B.

THE CREDIT GOES TO (IjlalHashmi)

Hi, does anyone know the answer to this question? I’m bad at geometry and I’m struggling to answer it.

Answers

Answer:

QT = 16

Step-by-step explanation:

ΔQRS ~ ΔQRT

In similar triangles, corresponding angles are in same ratio.

[tex]\frac{QS}{QR}=\frac{QR}{QT}\\\\\frac{25}{20}=\frac{20}{QT}[/tex]

Cross multiply,

QT * 25 = 20 * 20

[tex]QT =\frac{20*20}{25}\\\\QT = 4*4\\\\QT = 16[/tex]

Answer two questions about Equations A and B:
A. 3x – 1= 7
B. Зx=8
1) How can we get Equation B from Equation A?

Answers

Answer:

1. c) Add/ Subtract the same quantity to/from both sides

2) Yes

Step-by-step explanation:

Answer two questions about Equations A and B:

A. 3x – 1= 7

B. Зx = 8

1) To get Equation B from Equation A we need to take the following steps

A. 3x – 1 = 7

We add one to both sides

3x - 1 + 1 = 7 + 1

3x - 0 = 8

3x = 8

Option c is the correct option.

2) We are given Equations:

A. 3x – 1= 7

B. Зx = 8

3x - 1 = 7 = 3x = 8

3x = 7 + 1 = 3x = 8

3x = 8 = 3x = 8

Therefore, we can say, Yes, Equation A is equivalent to Equation B, hence they have the same solution.

58/360 in simplest form​

Answers

Answer:

29/180 is the answer for 58/360 in simplest form

I have $5$ different mathematics textbooks and $4$ different psychology textbooks. In how many ways can I place the $9$ textbooks on a bookshelf, in a row, if there must be a psychology textbook exactly in the middle, and there must be a mathematics textbook at each end?

Answers

Answer:

What

Step-by-step explanation:

You can’t have more than 2 combinations with £9

HELPP PLS ASAP I NEED HELP

Answers

Answer:

Option (b) second one is the correct answer

What is the 30th term of the linear sequence below?
-4,-1,2,5,8

Answers

Answer:

83

Step-by-step explanation:

a_1 = -4

a_2 = -1

a_2 - a1 = -1 - (-4) = 3

Each new term is 3 added to the previous term.

a_n = a_1 + 3(n - 1)

Test: Let's find the 4th term and see if we get a_4 = 5.

a_4 = a_1 + 3(4 - 1) = -4 + 3(3) = -4 + 9 = 5

We see that the 4th term really is equal to 5.

a_30 = a_1 + 3(30 - 1) = -4 + 3(29) = -4 + 87 = 83

what is the solution to the system of equations graphed below?​

Answers

I believe it would be (A) 0.-4

Please could someone help me?

Answers

Answer:

C

Step-by-step explanation:  6  23  17  16

15 is the highest value

Answer:

7:00 pm

Step-by-step explanation:

Mode - Result that appears the most

15 people said they preferred the time 7pm. 15 is the largest number out of the given data

So the mode is 7pm

Marc is building a rectangular wooden frame for his canvas. If the canvas is 7 feet long by 5 feet wide, what is the perimeter of the canvas?

Answers

Answer:

24 feet

Step-by-step explanation:

Use the perimeter formula, P = 2l + 2w, where l is the length and w is the width.

Plug in the values, and solve:

P = 2l + 2w

P = 2(7) + 2(5)

P = 14 + 10

P = 24

So, the perimeter of the canvas is 24 feet

A child weighs 44 lbs. The physician orders a medication for 10 mg/Kg/day. What do you give per day? (Use 0.45 for your conversion

Answers

Answer:

198 mg

Step-by-step explanation:

A child weighs 44 lbs.

Step 1

We convert the child's weight to kg

1 Ibs = 0.45kg

44 Ibs = x

Cross Multiply

1 Ibs × x = 44 Ibs × 0.45 kg

x = 44 Ibs × 0.45 kg/1 Ibs

x = 19.8 kg

Step 2

The physician orders a medication for 10 mg/Kg/day.

From the above statement, we know that:

10mg = 1 kg

Hence, what you give per day is calculated as:

1 kg = 10 mg

19.8kg = x

Cross Multiply

1 kg × x = 19.8kg × 10mg

x = 19.8kg × 10mg/1kg

x = 198mg

Therefore, what you will give per day to a child of 19.8kg(44 Ibs) is 198mg per day

Someone help please struggling on this answer thank you so much!

Answers

Answer:

-x+8=6

Step-by-step explanation:

x+5-1(2x)+(-1)(-3)=6

x+5-1-2x+3=6

ans (-x+8=6)

x+8

Jaclyn used the slope formula to find the slope of the line through the points given in the table.

A 2-column table with 2 rows. Column 1 is labeled x with entries 0, negative 3. Column 2 is labeled y with entries 4, 0.

1. Point 1: (0, 4); Point 2: (negative 3, 0). 2. Slope = StartFraction y 2 minus y 1 Over x 2 minus x 1 EndFraction = StartFraction 0 minus 4 Over negative 3 minus 0 EndFraction = StartFraction negative 4 Over negative 3 EndFraction. 3. Slope = four-thirds.

In which step did Jaclyn make an error?

Answers

Answer: She didn't make an error.

Step-by-step explanation:

She just didn't I got it right.

Answer:

answer is D

Step-by-step explanation:

Hi, does anyone know the answer to this question? I’m bad at geometry and I’m struggling to answer it.

Answers

The answer is (2) a translation

Please Mark BRAINLIEST!

Homework June 10th 2021 Jim

Answers

Problem 1

Part (a)

It is possible to find the equation of the line of best fit by hand, but it's much more efficient to use technology. That could mean a graphing calculator, an online tool, or some computer software installed. I'm going to use GeoGebra. Specifically, I'm using the "FitLine" command to find the regression line.

The equation of line of best fit is y = -2.45x + 11.83

There's not much to say in terms of steps, since it's basically a calculator problem. Doing this by hand would take a lot longer than it should be.

------------

Part (b)

Plug in x = 2.3 and evaluate

y = -2.45x + 11.83

y = -2.45*2.3 + 11.83

y = 6.195

Answer: 6.195

-------------

Part (c)

We'll compare the result from the previous part (6.195) to the y value in the table (6.2)

The difference is 6.2 - 6.195 = 0.005

This means the estimated value is off by 0.005 and this is an underestimate (since 6.195 is smaller than 6.2)

===================================================

Problem 2

To be perfectly honest, I'm not sure what's going on here. It seems like there's missing context to the problem. Perhaps a data table that got cut off or this is referring to a previous problem.

===================================================

Problem 3

Recall that y = mx+b is the slope intercept form

m = slope

b = y intercept

We simply read off the values of the given equation y = 0.404x - 5.18 to see that 0.404 is the slope and -5.18 is the y intercept

The slope tells us that each time x goes up by 1, y will increase by an estimated amount of 0.404; this represents the unit rate or speed. In this context, it means the height goes up by about 0.404 cm per day. This is an estimated value because the regression line itself is a collection of estimated points.

The y intercept is where the graph crosses the y axis. It always occurs when x = 0. The x refers to the day number. Day 0 is basically the starting day. So the y intercept being -5.18 means the estimated height is -5.18 cm on the starting day. At first glance, it might not seem possible to have a negative height. But simply think of negative heights as below ground level, much like how the negative y values are below the x axis.

In other words, the plant is estimated to start off at about 5.18 cm below ground level. This is a reasonable assumption because the seed is buried into the ground (assuming at this level more or less) and it then grows upward.

If cos theta = 0.8, find 1 / sin (pi/2 - theta)

Answers

Answer:

J

Step-by-step explanation:

Using the cofunction identity

cosθ = sin([tex]\frac{\pi }{2}[/tex] - θ )

Then

[tex]\frac{1}{sin(\frac{\pi }{2}-0) }[/tex]

= [tex]\frac{1}{cos0}[/tex]

= [tex]\frac{1}{0.8}[/tex]

= 1.25 → J

The value of 1/sin([tex]\frac{\pi }{2}[/tex] - θ) is 1.25.

What are four quadrants of trigonometry?

The coordinate axes divide the plane into four quadrants, labelled first, second, third and fourth as shown. Angles in the third quadrant, for example, lie between 180 degrees and 270 degrees.

Given

1/cosθ = 0.8

1/sin([tex]\frac{\pi }{2}[/tex] - θ) =?

By using quadrants in trigonometry

we know that sin([tex]\frac{\pi }{2}[/tex] - θ) = cosθ

= 1/sin([tex]\frac{\pi }{2}[/tex] - θ)

= 1/cosθ

= 1/0.8

= 1.25

1/sin([tex]\frac{\pi }{2}[/tex] - θ) = 1.25

Hence, the value of 1/sin([tex]\frac{\pi }{2}[/tex] - θ) is 1.25.

Learn more about four quadrants of trigonometry here

https://brainly.com/question/21864197

#SPJ2

2) Simplify: 4/3+3/5+-3/5+-11/3​

Answers

Answer:

[tex]\frac{4}{3}+\frac{3}{5}+-\frac{3}{5}+-\frac{11}{3}[/tex][tex]=\frac{4}{3}-\frac{11}{3}[/tex][tex]\frac{a}{c}\pm \frac{b}{c}=\frac{a\pm \:b}{c}[/tex][tex]=\frac{4-11}{3}[/tex][tex]\:4-11=-7[/tex][tex]=\frac{-7}{3}[/tex][tex]\:fraction\:rule}:\quad \frac{-a}{b}=-\frac{a}{b}[/tex][tex]\frac{-a}{b}=-\frac{a}{b}[/tex][tex]\frac{-7}{3}[/tex][tex]=-\frac{7}{3}[/tex]---------------------[tex]\boxed{\boxed{\underline{\textsf{\textbf{hope it helps}}}}}[/tex][tex]\boxed{\boxed{\underline{\textsf{\textbf{have a great day!}}}}}[/tex]

Which is equivalent to
1/4x

Answers

Answer:

1/4x is basically x divided by 4/multiplied by 1/4

this can be applied in examples such as:

1/4(2) = 0.5

etc.

Step-by-step explanation:

i hope this helped, if not, please leave a comment specifically requiring what exactly you are stuck with.

Answer:

 1/8x+1/8x

Step-by-step explanation:

Question 3 of 10
In the diagram below, Ed is parallel to Xy. What is the value of x?

Answers

The answer is C because it interior angles and 180-105 =75

The value of y will be 75°. The correct option is C.

What are lines and angles?

Straight lines with little depth or width are present. You will learn about a number of lines, including transversal, intersecting, and perpendicular lines.

A figure called an angle is one in which two rays originate from the same point. In this area, you could also encounter contrasting and related viewpoints.

Given that the two parallel lines are DE and XY. The angle y will be calculated as,

y = 180 - 105

y = 75°

Hence, the measure of the angle y is 75°

To know more about lines and angles follow

https://brainly.com/question/28769265

#SPJ2

A circle has a radius of 18 ft and an arc length of 117. What is the measure of the central angle of
the circle?

Answers

20akakdjhajaksakakka

A mistake was made in the steps shown to simplify the expression. Which step includes the mistake?
1 + 92
5
+ -10% = 2
Step 1: = 1 + 32 + 10 = 2
Step 2:=
1* ° + 10 = 2
1 + 10 = 2
Step 4: = 2 + 10 = 2
Step 5: = 12 = 2
Step 6: = 6
Step 3:=
O A. Step 4
doorn
O B. Step 5
O C. Step 1
D.
Step 3
M

Answers

Answer:

step 4

Step-by-step explanation:

Answer:

Step  4

Step-by-step explanation:

[tex]\frac{1 + 3^2}{5} + |-10| \div 2\\\\Step \ 1 : \frac{1 + 3^2}{5} + 10 \div 2\\\\step \ 2 : \frac{1 + 9}{5} + 10 \div 2 \\\\Step \ 3 : \frac{10}{5} + 10 \div 2\\\\Step\ 4: 2 + 10 \div 2\\\\Step \ 5 : 2 + 5 \ \ \ \ \ \ \ \ \ \ \ \ \ [ \ given : 12 \div 2 \ ]\\\\Step \ 6 : 7[/tex]

Other Questions
Who became the emperor/leader of Japan after the emperor Go- Daigo was defeated during the Japanese civil war?A- Kublai Khan B- Ashi Hoto C- AshikaganD- Hirohito Ponds and lakes are part of marine ecosystem.True or false ? g(x) = 3x -1. Find x such that g(x)= 20.g(x)= 3x+1. Find x such that g(x)= 22.g(x)= 2x-5. Find g(1/2) and g(a)=99.g(x)=x+5/2. Find g(3), g(0), g(-3) and g(x)=0Please solve this quickly. I need it right now. No silly answer will not be allowed. PLEASE HELP!!!WILL MARK BRAINLIEST!!!!If the measure of 0 is 90, then angle B=____Multiple choice!Thank you!!! An animal dies in mud, decomposes, and leaves an imprint, which is later filled by sediment to form a solid copy of theoriginal organism.How is this fossil different from fossils of preserved remains?O Only hard tissue is preserved.O Hard and soft tissues are preserved.O Only soft tissue is preserved.O There is little change to the preserved organism. PLEASE I HAVE BRAINLY PLUS AND IT NOT WORKING SEND HELLP A compound formula is found to contain 31.42% sulfur, 31.25% oxygen, and 37.23% fluorine by mass. Calculate the molecular formula if the molar mass is 204 g/mol Marcus is testing a hair sample from the telogen stage. What is likely to be TRUE about this sample? A. There will be several medullas present. B. There will be mitochondrial DNA present. C. There will be nuclear DNA present. D. There will be very little DNA present. Which quadrilateral makes this statement true? Quadrilateral ABCD _____ A. B. C. D. Shawn thinks that helium prices next year might increase by 10%. He plans to buy 7 small balloons and x large balloons. He creates the following equation to find the total price for next years helium:P Suppose the government imposes a 20-cent tax on the sellers of artificially-sweetened beverages. The tax would shift a. demand, lowering the equilibrium price and raising the equilibrium quantity in the market for artificially sweetened beverages. b. supply, raising the equilibrium price and lowering the equilibrium quantity in the market for artificially sweetened beverages. c. supply, lowering the equilibrium price and raising the equilibrium quantity in the market for artificially sweetened beverages. d. demand, raising both the equilibrium price and quantity in the market for artificially sweetened beverages. Some triangles can have more than one obtuse angle. A. True B. False Classify each of the following as a Strong acid (sa) or a Weak acid (wa) and indicate how each should be written in aqueous solution. 1. hydrobromic acid2. hydrochloric acid 3. carbonic acidWrite a net ionic equation for the reaction that occurs when aqueous solutions of hydrochloric acid and barium hydroxide are combined. expand and simplify 2(3x - 5) + 3(x + 3)can someone do this ASAP pls with no links :) Please help!How did the narrator feel when he heard the strange cries, saw the wild eyes, and heard the growl from the side of the canal? Use evidence from the story to support your answer.Why wont the narrator return to Xochimilco, even though the canals are safe again? Use evidence from the text to support your answer.(passage: page 7, questions: page 11) Select the correct answer.On a trip to a wildlife sanctuary, Nathan found a piece of rotting log. Helog. He took a sample from the log to examine In his laboratory. The organismhe found was eukaryotic and could not photosynthesize. It also had a cell wall.What type of organism was most likely feeding on this sample?O A. plant-like protistO B. animal-like protistO C. fungus-lIke protist D.archaeaO E.bacteria Ill give someone brainliest!!! Help!!! Which of the following is the equation of a line perpendicular to the line y = - 1/3 * x + 1 passing through the point (2, 7) ? A. 3x - y = 1 B. - 3x - y = 1 C. 3x - y = - 1 D. - 3x + y = - 1 The area of the base is 15cm. The height is 6 cm. What is the volume of the rectangular prism? The image shows the right-hand rule being used for a current-carrying wire.An illustration with a right hand with fingers curled and thumb pointed up.Which statement describes what the hand shows?When the current flows down the wire, the magnetic field flows out on the left side of the wire and in on the right side of the wire. When the current flows up the wire, the magnetic field flows out on the left side of the wire and in on the right side of the wire. When the current flows down the wire, the magnetic field flows in on the left side of the wire and out on the right side of the wire. When the current flows up the wire, the magnetic field flows in on the left side of the wire and out on the right side of the wire. Complete the text adding suffixes, prefixes or the both with the root words given in the parenthesis. 0.5x10-5 (Adequate) power supply is doing harm to our economy. It hampers (b) (produce) in mills and factories. Farmers cannot get as much electricity as they need for (c) (irrigate) purposes. Moreover there are some areas where rural (d) (electrify) scheme has not been (e) (implement). Desired benefit is not (f) (obtain) (implement) of the scheme unless (h) after the (g) (pilfer) of electricity is stopped. (i) electricity is one of the major reasons of its (j) (short). (use) of